The United States Department of Agriculture (USDA) found that the proportion of young adults ages 20–39 who regularly skip eating breakfast is 0.238 . Suppose that Lance, a nutritionist, surveys the dietary habits of a random sample of size =500 of young adults ages 20–39 in the United States.Use a normal approximation to find the probability that the number of individuals, , in Lance's sample who regularly skip breakfast is greater than 124 .(>124)= (Round to 3 decimal places)

Answers

Answer 1

Answer

The answer is 0.300

Problem Statement

We are asked to find the probability that the number of individuals in a survey of 500 people would skip breakfast given that the proportion of people who skip breakfast, in general, is 0.238.

Method

- The proportion of people greater than 124 out of 500 is easily gotten to be:

[tex]\begin{gathered} p>\frac{124}{500} \\ p>0.248 \end{gathered}[/tex]

- We now need to know the probability that the proportion of people that skip breakfast would be greater than 0.248.

- To calculate this probability, we need to find the Z-score associated with this value. This is a good way to approximate the probability because the number of people in the survey is well above 30 and we have been told to apply a normal approximation.

- Once we have the Z-score associated with this proportion of 0.248 in relation to the general population proportion statistic of 0.238, we can then convert the Z-score into a probability using a Z-score calculator or a Z-table.

- If the Z-score is "z", then, the probability we are looking for on the Z-score table or calculator is P(x > z).

- Thus, we can solve the question using the following steps:

1. Calculate the Z-score using the formula below:

[tex]\begin{gathered} z=\frac{p-p_0}{\sqrt[]{\frac{p_0(1-p_0)}{n}}} \\ \\ \text{where,} \\ p=\text{sample proportion} \\ p_0=\text{population proportion} \\ n=\text{ Total number of people in the survey} \end{gathered}[/tex]

2. Convert the Z-score into probability

Implementation

Step 1: Calculate the Z-score:

[tex]\begin{gathered} p=0.248,p_0=0.238 \\ \\ z=\frac{0.248-0.238}{\sqrt[]{\frac{0.238(1-0.238)}{500}}} \\ \\ z=\frac{0.01}{0.019045} \\ \\ z=0.5251 \end{gathered}[/tex]

2. Convert the Z-score into probability:

Using the Z-score calculator, we have:

Because we are asked to find the probability that the number of people who skipped breakfast is greater than 124, the correct probability here is P(x > Z).

Thus, the probability that the number of individuals that skipped breakfast is greater than 124 is 0.29977 ≅ 0.300 (To 3 decimal places)

Final Answer

The answer is 0.300.

The United States Department Of Agriculture (USDA) Found That The Proportion Of Young Adults Ages 2039

Related Questions

Choose the median for the set of data. 99 95 93 92 97 95 97 97 93 97 a. 7b. 95.5 c. 96d. 97

Answers

The median is the middle of a sorted list of number. So, we need to place the number in value order, that is,

[tex]92,93,93,95,95,97,97,97,97,99[/tex]

then, the middle is between the 5th and 6th number:

then, we need to find the mean value of these numbers. So, the median is

[tex]\text{ median=}\frac{95+97}{2}=96[/tex]

Therefore, the answer is option C.

Find the slope and the y-intercept of the line. 4x + 2y= -6 Write your answers in simplest form. Undefined 08 slope: . X ? y -intercept: 0

Answers

Transform equation form Ax + By = C

to y = ax + b

THen

4x + 2y = -6

A= 4. B= 2. C= -6

y = (-A/B)•x +(D/B)

y= (-4/2)•x + (-6/2)

y = -2x -3

Therefore in new equation

Slope a = -2

Y intercept b = -3

A school librarian would like to buy subscriptions to 7 new magazines. Her budget however, will allow her to buy only 4 new subscriptions. How many different groups of 4 magazines can she chose from the 7 magazines?

Answers

The number of groups of 4 magazines can she choose from the 7 magazines is 35

Total number of magazines that school librarian would like to buy subscription = 7 magazines

The number of subscription that she can afford = 4 new subscription

The different groups of 4 magazines can she choose from the 7 magazines = [tex]7C_4[/tex]

The combination is the method of selecting a particular items or objects from the group of collection. The combination can also be defined as the number of possible arrangement from the collection.

Then the value of

[tex]7C_4[/tex] = 7! / 4!(7 - 4)!

= 7! / (4! × 3!)

= (7 × 6 × 5 × 4!) / (4! × 3!)

= (7 × 6 × 5) / 3!

= (7 × 6 × 5) / 3 × 2 × 1

= 210 / 6

= 35

Hence, the number of groups of 4 magazines can she choose from the 7 magazines is 35

Learn more about combination here

brainly.com/question/28720645

#SPJ1

h(x) = 10x - x^2 find h(4)

Answers

We have the following expression

[tex]h(x)=10x-x^2[/tex]

In our case x is equal to 4, then, we will evalute the given function h(x) when x is 4. It yields,

[tex]h(4)=10(4)-(4)^2[/tex]

which gives

[tex]\begin{gathered} h(4)=40-16 \\ h(4)=24 \end{gathered}[/tex]

Therefore, the asnswer is h(4)=24

lines m and n are paralle. Find the measures of angles x, y, and z in the figure

Answers

Explanation

From the image, angle x and 65 degrees form angles on a straight line. We will recall that the sum of angles on a straight line sums up to 180 degrees.

Therefore,

[tex]\begin{gathered} x+65^0=180^0 \\ x=180^0-65^0 \\ x=115^0 \end{gathered}[/tex]

Angle y and 65 degrees form alternate angles, we will recall that alternate angles are equal

Therefore,

[tex]y=65^0[/tex]

Angle x and angle z form corresponding angles, we will recall that corresponding angles are equal.

Therefore,

[tex]z=115^0[/tex]

Answer:

[tex]x=115^0,y=65^0,z=115^0[/tex]

find the inverse of each given function f(x)=4x-12f^-1(x)=______x+______

Answers

The original function is f(x) = 4x - 12...

to find the inverse function, we need to solve it for x:

f(x) = 4x - 12

f(x) + 12 = 4x

(f(x) + 12)/ 4 = x

f(x)/4 + 3 = x

if we change now f^-1(x) for x and x for f^-1(x):

x/4 + 3 = f^-1(x)

f^-1(x) = x/4 + 3

f^-1(x) = (1/4)x + 3

Answer:

which statements and reason complete steps 3 , 4 and 6 of the proof ?

Answers

Pythagoras theorem

Statement 1:

ΔABC ≅ ΔCBD ≅ ΔACD

Reason: Given

_________________________________

Statement 2:

b/c = y/b; a/x = x/a

Reason: corresponding sides of similar triangles are proportional

(we want to have to have in the next statement that b² = cy; a² = cx

and proportionality is usually represented as fractions, if we observe the figure, the fractions of this statement correspond to the division of similar sides of the triangles)

________________________

Statement 3:

b² = cy; a² = cx

Reason: cross product property

(if we multiply both sides of b/c = y/b by b, we obtain b² = cy, and if we do the same for a/x = x/a we obtain a² = cx, since we are multiplying, it is called product, then, the option that best fit this field is cross product property)

_______________________________

Statement 4:

a² + b² = cx + cy

Reason: addition property of equality

(we want to prove that a² + b² = c², from the previous statement we can add both equalities so we obtain a² + b² , which is nearer to the conclusion we want to prove)

____________________

Statement 5:

a² + b² = c(x + y)

Reason: factor

(we find the common factor of cx and cy, it is c, then cx + cy = c(x + y))

___________________________

Statement 6:

c = x + y

Reason: Segment addition postulate

(we almost have the conclusion in the previous statement except for the (x + y) of the right part of the equality, since in the figure we observe that c = x + y, then we can use it to replace (x + y))

___________________________

Statement 7:

a² + b² = c²

Reason: substitution

(we substitute c by (x + y) of the statement 5)

looking to recieve help with finding the vertex of the parabola.

Answers

In this case, we'll have to carry out several steps to find the solution.

Step 01:

Data

f(x) = - 2x² + 4x + 2

Step 02:

y = ax² + bx + c

a = -2

b= 4

c = 2

vertex of the parabola equation

[tex]yv\text{ =- }\frac{b^{2}-4ac}{4a}[/tex][tex]\begin{gathered} yv\text{ = -}\frac{4^2-4\cdot(-2)\cdot(2)}{4\cdot(-2)} \\ yv\text{ = -}\frac{(16+16)_{}}{-8} \end{gathered}[/tex]

yv = (- 32) / (- 8)

yv = 4

[tex]xv\text{ = -}\frac{b}{2a}[/tex][tex]\begin{gathered} xv\text{ =- }\frac{4}{2(-2)} \\ xv\text{ = }\frac{-4}{-4} \end{gathered}[/tex]

xv = 1

Vertex:

(xv , yv ) = (1 , 4 )

The answer is:

The vertex of the parabola is (1 , 4)

write the equation for this line in slope intercept form.y= ? × + __ a) -4 b) 2c) -2 d) -1/2

Answers

we know that

the equation in slope intercept form is equal to

y=mx+b

In this problem

we have

b=-4 ------> because the y-intercept is (0,-4)

Find the slope

we need two points

we take

(-2,0) and (0,-4)

so

m=(-4-0)/(0+2)

m=-4/2

m=-2

therefore

y=-2x-4

The expression 12x+6 can be used to describe a sequence algebraically. Which of the following could be the first five numbers in this sequence?A. 18, 36, 54, 72, 90B. 6, 12, 18, 24, 30C. 18, 30, 42, 54, 66D. 6, 18, 24, 36, 42

Answers

We need to find the first five numbers of a sequence determined by the expression:

[tex]12x+6[/tex]

Notice that each time we increase the value of x by 1 unit, we add 12 to the previous result. Thus, subsequent terms in the sequnce differ by 12 units.

From the options, the only one with all the terms differing by 12 units is the beginning at x=1:

[tex]\begin{gathered} x=1:12(1)+6=18 \\ \\ x=2:12(2)+6=30 \\ \\ x=3:12(3)+6=42 \\ \\ x=4:12(4)+6=54 \\ \\ x=5:12(5)+6=66 \end{gathered}[/tex]

Therefore, the answer is: C. 18, 30, 42, 54, 66

Suppose a sample of 879 new car buyers is drawn. Of those sampled, 288 preferred foreign over domestic cars. Using the data construct a 95% confidence interval for the population proportion of new car buyers who prefer for foreign cars over domestic cars. Round your answers to three decimal places

Answers

To find the confidence interval for a proportion, we use the following formula:

[tex]Confidence\text{ }interval=p\pm z\cdot\sqrt{\frac{p(p-1)}{n}}[/tex]

Where:

p is the sample proportion

z the chosen z-value

n sample size

Since we want to make a confidence interval of 95%, we need to use z = 1.96. The sample size is n = 879.

We can use cross multiplication to find p, which is the percentage of the total sample size that preferred foreign cars:

[tex]\begin{gathered} \frac{879}{288}=\frac{100\%}{x} \\ . \\ x=100\%\cdot\frac{288}{879} \\ . \\ x=32.765\% \end{gathered}[/tex]

p is the proportion in decimal, we need to divide by 100:

[tex]p=\frac{32.765}{100}=0.32765[/tex]

Now, we can use the formula:

[tex]Confidence\text{ }interval=0.32765\pm1.96\sqrt{\frac{0.32765(1-0.32765)}{879}}=0.32765\pm0.031028[/tex]

[tex]\begin{gathered} Lower\text{ }endpoint=0.32765-0.031028=0.296616 \\ Upper\text{ }endpoint=0.32765+0.031028=0.35867 \end{gathered}[/tex]

Thus, the answer is:

Lower endpoint: 0.297

Upper endpoint: 0.359

Can someone please help me with this math, thank you

Answers

Given data:

The given growth rate is r=7.8%=0.078.

The final number of bacteias in terms of initial is P'=2P.

The expression for the bacterias growth rate is,

[tex]P^{\prime}=P(1+r)^t[/tex]

Substitute the given values in the above expression.

[tex]\begin{gathered} 2P=P(1+0.078)^t \\ 2=(1.078)^t \\ \ln (2)=t\ln (1.078) \\ t=\frac{\ln(2)}{\ln(1.078)} \\ =9.23\text{ hours} \end{gathered}[/tex]

Thus, after 9.23 hours population of the bacterias doubled.

$480 invested at 15% compounded quarterly after a period of six years

Answers

Answer: $1161

Step-by-step explanation: The equation for compound interest is A=P(1+r/n)^n*t. P is the principal, in this case, being $480 originally invested, r is the rate, in this case being 15% or 0.15, and n is 4 because it is compounded quarterly. t is 6 because the period invested is 6 years. A=480(1+0.15/4)^4*6. This can simplify to 480(1.0375)^24, which equals approximately $1161 dollars. If the question requires to the tenths, it is $1161.3, and for the hundredths, $1161.33.

Height: Suppose you are 5 feet 8 inches tall. Give your height in meters and centimeters.For example, "9'2" = 2.8 meters = 2 meters and 80 centimeters."You are meters andcentimeters.

Answers

Height is 5 feet 8 inches.

1 feet is 12 inches. So,

(5*12) + 8 = 68 inches

Now, let's convert to meters.

We know:

1 inch = 0.0254 meters

So, 68 inches would be:

68 * 0.0254 = 1.7272 meters

We would need to convert the fractional part (excess of 1, which is 0.7272) to cm.

We know:

1 m = 100 cm

So,

0.7272 m is:

0.7272 * 100 = 72.72 cm

Hence,

The answer is:

1 meters and 73 centimeters (rounded to neaerest cm)

PLSSS ANSWER ASAP PLS!!!! Solve y3 = 27.A. y = 9 B. y = 3 y= 3 C. y = 3 D. y = 5.2

Answers

[tex]y^3=27[/tex][tex]y=\sqrt[3]{27}[/tex][tex]undefined[/tex]

A petrified stump that is 4 ft tall casts a shadow that is 2 ft long. Find the height of a tent that casts a 5 ft shadow

Answers

The petrified stump is 4 ft tall and cast a shadow that is 2 ft long .

2 ft shadow has a 4 ft height

5 ft shadow will have ? height

cross multiply

[tex]\begin{gathered} \text{height of tent = }\frac{5\times4}{2} \\ \text{height of tent = }\frac{20}{2} \\ \text{height of tent = 10 ft} \end{gathered}[/tex]

A bookstore spent $241 to send a group of students to a readingcompetition. Each student who won was given a $5 gift certificate. Anda personalized bookmark that cost $2. Included in the $241 was $45 forthe salary of a staff member who accompanied the students to thecompetition. How many students won prizes?

Answers

A bookstore spent $241 to send a group of students to a reading

competition. Each student who won was given a $5 gift certificate. And

a personalized bookmark that cost $2. Included in the $241 was $45 for

the salary of a staff member who accompanied the students to the

competition. How many students won prizes?

Let

x -----> number of students that won prizes

we have that

the equation that represents this situation is

241=(5+2)x+45

241=7x+45

solve for x

7x=241-45

7x=196

x=28

therefore

28 students won prizes

Work out the rage 51,38,48,36,39,40,39,47

Answers

The range of the given data set will be 15.

What is the range?When the sample maximum and minimum are subtracted, the range of a collection of data is the difference between the greatest and lowest values. It uses the same units as the data to express itself.Find the biggest observed value of a variable (the maximum) and subtract the smallest observed value to determine the range (the minimum).The range is the range of values, from lowest to highest. Example: The lowest value in 4, 6, 9, 3, and 7 is 3, while the highest value is 9. The range is therefore 9 - 3 = 6.

So, the range of the given data:

In increasing order: 36, 38, 39, 39, 40, 47, 48, 51

The range will be:

51 - 3615


Therefore, the range of the given data set will be 15.

Know more about the range here:

https://brainly.com/question/2264373

#SPJ1

Correct questions:

Work out the range 51,38,48,36,39,40,39,47

2 5/6 divided by 1 3/4
Find the quotient. If possible, rename the quotient as a mixed number or a whole number. Write your answer in simplest form, using only the blanks needed.

Answers

The quotient is 113/21.

What is a mixed number?

It is formed by combining three parts a whole number, a numerator and a denominator. Here, the numerator and denominator are  a part of the proper fraction that makes the mixed number. These are also known as mixed fractions. It contains both an integer or a whole number. A mixed fraction or number is therefore a product of a whole number and a proper fraction.

2 5/6 = (2·6 +5)/6 = 17/6

1 3/4 = (1·4 +3)/4 = 7/4

Here 17/6 is dived by 7/4,we get

(17/6) ÷ (7/4) = (17/6) × (4/7) = (17×4)/(6×7) = (17×2)/(3×7) = 34/21

Here 34/21 is converted into a mixed number.

34/21 = (21 +13)/21 = 1 13/21

Therefore, the quotient is 113/21

To know more about mixed number, visit:

brainly.com/question/24137171

#SPJ1

Use the given triangle to fill in the blank.bCasin BB

Answers

We can apply trigonometric ratios, on this case we ned to use sine

[tex]\sin (\alpha)=\frac{O}{H}[/tex]

Where alpha is th reference angle, O the opposite side from the reference angles and H the hypotenuse of the triangle

On our case O is b and H is c, then replacing

[tex]\sin (B)=\frac{b}{c}[/tex]

then sinB is b/c, then right option is first

Use synthetic division to find the result when x³ + 3x² - 6x + 20 is divided by
x + 5.

Answers

Answer:

[tex]x^{2} + 8x + 34 + \frac{190}{x-5}[/tex]

Step-by-step explanation:

Which trig equation should be used to solve for x?

Answers

Solution:

To find the appropiate trigonometric formula.

we know that,

for the right angle triangle, we have that,

[tex]\sin \theta=\frac{opposite\text{ side}}{hypotenuse}[/tex]

The side opposite to the right angle is hypotenuse, the side opposite to the angle theta is opposite side and the other side is adjacent side.

Also we have,

[tex]\cos \theta=\frac{adjacent\text{ side}}{hypotenuse}[/tex][tex]\tan \theta=\frac{opposite\text{ side}}{adjacent\text{ side}}[/tex]

Using this we get,

[tex]\sin 37\degree=\frac{x}{12}[/tex]

Answer is:

[tex]\sin 37\degree=\frac{x}{12}[/tex]

What is the area of the rectangle whose coordinates are at A(-1,4), B(3, 2), Clo,-4) and D(-4,-2) (Round to the nearest whole number.)

Answers

Answer:

Explanation:

The area of the rectangle with the given coordinates is:

[tex]undefined[/tex]

Please help me with this question:Graph the function F(x) = x^2 + 4x - 12 on the coordinate plane by finding the important points below.Be sure to show all steps in your calculations.(a)What are the x-intercepts?(b)What is the y-intercept?(c)What is the maximum or minimum value?(d)Use the points to graph the function.

Answers

Given the function:

[tex]f(x)=x^2+4x-12[/tex]

Let's graph the function.

Let's find the following:

• (a). x-intercepts:

The x-intercepts are the points the function crosses the x-axis.

To find the x-intercepts substitute 0 for f(x) and solve for x.

[tex]\begin{gathered} 0=x^2+4x-12 \\ \\ x^2+4x-12=0 \end{gathered}[/tex]

Factor the left side using AC method.

Find a pair of numbers whose sum is 4 and product is -12.

We have:

6 and -2

Hence, we have

[tex]\begin{gathered} (x+6)(x-2)=0 \\ \\ \end{gathered}[/tex]

Equate the individual factors to zero and solve for x.

[tex]\begin{gathered} x+6=0 \\ Subtract\text{ 6 frm both sides:} \\ x+6-6=0-6 \\ x=-6 \\ \\ \\ x-2=0 \\ Add\text{ 2 to both sides:} \\ x-2+2=0+2 \\ x=2 \end{gathered}[/tex]

Therefore, the x-intercepts are:

x = -6 and 2

In point form, the x-intercepts are:

(x, y) ==> (-6, 0) and (2, 0)

• (b). The y-intercept.

The y-intercept is the point the function crosses the y-axis.

Substitute 0 for x and solve f(0) to find the y-intercept:

[tex]\begin{gathered} f(0)=0^2+4(0)-12 \\ \\ f(0)=-12 \end{gathered}[/tex]

Therefore, the y-intercept is:

y = -12

In point form, the y-intercept is:

(x, y) ==> (0, -12)

• (c). What is the maximum or minimum value?

Since the leading coefficient is positive the graph will have a minimum value.

To find the point where it is minimum, apply the formula:

[tex]x=-\frac{b}{2a}[/tex]

Where:

b = 4

a = 1

Thus, we have:

[tex]\begin{gathered} x=-\frac{4}{2(1)} \\ \\ x=-\frac{4}{2} \\ \\ x=-2 \end{gathered}[/tex]

To find the minimum values, substitute -2 for x and solve for f(-2):

[tex]\begin{gathered} f(-2)=(-2)^2+4(-2)-12 \\ \\ f(-2)=4-8-12 \\ \\ f(-2)=-16 \end{gathered}[/tex]

Therefore, the minimum value is at:

y = -16

Using the point form, we have the minimum point:

(x, y) ==> (-2, -16).

• (d). Use the points to plot the graph.

We have the points:

(x, y) ==> (-6, 0), (2, 0), (0, -12), (-2, -16)

Plotting the graph using the points, we have:

Find value of x. Math 80 I know it’s something to do with sine right?

Answers

Given

To find the value of x.

Explanation:

It is given that,

[tex]\theta=34\degree[/tex]

Then,

[tex]\begin{gathered} \sin34\degree=\frac{x}{29} \\ 0.55919\times29=x \\ x=16.21659 \\ x=16.22 \end{gathered}[/tex]

Hence, the value of x is 16.22.

Which expression is equivalent to 6x + 7- 12.2 - (32 + 2) - x?(A)7x - 28B7x - 21©5x - 28D5x - 21please hurry

Answers

[tex]\begin{gathered} 6x+7-12\cdot2-(3^2+2)-x=6x-x+7-24-(9+2) \\ =5x+7-24-11 \\ =5x-28 \\ \\ \text{ The answer is 5x-28!} \end{gathered}[/tex]

Which inequality represents all values of x for which the quotient below is defined? (Division)

Answers

We want to calculate the following quotient

[tex]\frac{\sqrt[]{28(x-1)}}{\sqrt[]{8x^2}}[/tex]

Note that using properties of radicals, given non zero numbers a,b we have that

[tex]\frac{\sqrt[]{a}}{\sqrt[]{b}}=\sqrt[]{\frac{a}{b}}[/tex]

So, using this fact, our quotient becomes

[tex]\sqrt[]{\frac{28(x-1)}{8x^2}}[/tex]

As we are taking the square root, this opearation is only valid if and only if the expression inside the square root is a non negative number. That is, we must have that

[tex]\frac{28(x-1)}{8x^2}\ge0[/tex]

As this is a quotient, we should also that the quotient is defined.

To understand this last point, we should make sure that we are not dividing by 0. So first, we want to exclude those value s of 0 for which the denominator becomes 0. So we have the following auxiliary equation

[tex]8x^2=0[/tex]

which implies that x=0.

So, the second quotient is always defined whenever x is different from 0. However, assuming that x is not 0 we want to find the value of x for which

[tex]\frac{28(x-1)}{8x^2}\ge0[/tex]

To start with this problem, we solve first the equality. So we have

[tex]\frac{28(x-1)}{8x^2}=0[/tex]

since x is not 0, we can multiply both sides by 8x², so we get

[tex]28(x-1)=0\cdot8x^2=0[/tex]

If we divide both sides by 28, we have that

[tex]x-1=\frac{0}{28}=0[/tex]

now, by adding 1 on both sides we get that

[tex]x=1[/tex]

so, whenever x=1, we have that the quotient inside the radical becomes 0.

Now, we will solve the inequality, that is

[tex]\frac{28(x-1)}{8x^2}>0[/tex]

Note that on the left, we are mostly dividing two expressions. Recall that the quotient of two expressions is positive if and only if both expressions have the same sign.

Note that the expression

[tex]8x^2[/tex]

is the product of number 8 (which is positive) with the expression x², which is also always positive for any value of x. This means that the expression 8x² is always positive.

So, taking this into account, we should focus on those values of x for which the numerator is positive, as the denominator is always positive. So we end up with the following inequality

[tex]28(x-1)>0[/tex]

If we divide both sides by 28 we get

[tex]x-1>\frac{0}{28}=0[/tex]

So, if we add 1 on both sides, we get

[tex]x>1[/tex]

So, whenever x is greater than 1, the expression inside the radical is positive.

This means that the original quotient is defined whenever x=1 and whenever x>1. Thus, we would have

[tex]x\ge1[/tex]

Use the graph to find the slope and y-intercept of the line. Compare the values to the equation y= -3x+ 1

Answers

The y-intercept is at the point where the line cut the y-axis.

Hence, the y-intercept is 1

[tex]\begin{gathered} \text{Slope}=\frac{y_2-y_1}{x_2-x_1} \\ m=\frac{3-0}{-1-0} \\ m=\frac{3}{-1}=-3 \end{gathered}[/tex]

Hence, the slope is -3

Comparing the values to the equation y = =-3x +1, the equation is valid for the line.

A customer wants to leave a 15% tip. The bill was $35. How much should the customer leave as a tip?

Answers

The customer wants to leave 15% tip, if the bill is $35, then the tip is

[tex]=15\text{ \% of 35}[/tex][tex]=\frac{15}{100}\times\text{ \$35}[/tex][tex]=\text{ \$5.25}[/tex]

Therefore, the customer should leave $5.25 as a tip.

A chemist is using 383 milliliters of a solution of acid and water, If 17.3% of the solution is acid, how many milliliters of acid are there? Round your answer to the nearest tenth.

Answers

A chemist is using 383 milliliters of a solution of acid and water.

If 17.3% of the solution is acid, how many milliliters of acid are there?

We basically need to calculate 17.3% of 383 milliliters.

[tex]\begin{gathered} acid=17.3\%\: of\: 383\: mL \\ acid=\frac{17.3}{100}\times383 \\ acid=0.173\times383 \\ acid=6.3\: mL \end{gathered}[/tex]

Therefore, the solution has 6.3 milliliters of acid.

Other Questions
A water molecule consists of an oxygen atom with two hydrogen atoms bound to it. The angle between the two bonds is 106. If the bonds are 0.10nm long, where is the center of mass of the molecule? which amazon audience can be used to reach shoppers who recently viewed or purchased items in a specific category or sub-category? 16. Solve this system: y = 3x+1 y= 5x-3 3. *Which of the following equations has x intercepts at 4 and -2? (A) y = 3x^2 - 10x - 8 (B) y = x^2 + 2x - 8 (C) y = 3x^2 2x 8 (D) y = x^2 - 2x - 8 please help me!!!!!! If 8 more than twice a number is 14, what is 8 times the number? Simplify each expression by distributing8(x + 5) Judy is buying 6 pint of ice cream for her party at $3.45 each if she has a $20 bill does she have enough to buy the ice cream The equation ^2 4 4^2 + 13 = 0 will produce a hyperbola. How can we tell by simply observing the equation?In what directions do the branches of this hyperbola open? How do you know? Explain. Sketch a graph of this hyperbola, clearly indicating how you have determined thekey characteristics (center, vertices, eccentricity, foci). Give the domain and range of this hyperbola. Graph the equation on a coordinate grid:y=-1/2x How to write a rule for the nth term of the geometric seq A multivitamin tablet contains 12.5mg of calcium how much calcium does a bottle of 40 tablets contain write your answer in grams calculate the mass solute and the mass solution when the mass solvent is 315.2g and the mass percent is 15.3 %. 16 The water level of a river was measured each day during a two-week period. The graph models the linear relationship between the water level of the river in feet and the number of days the water level was measured. Water Level of River 28 20 16 Water Level (0) 3 4 6 8 10 12 14 Number of Days Which statement best describes the y-intercept of the graph? what value of y will make the equation true? 34 y = 34 Please help me with the last part of the question thanks solve for x 3/5x-1/3x =4 help meeeeeeeeeeeeee pleaseeeeeeeeeee rn rnnnn!!!!!!!!!!!!!!!!!!!!!!!!!!!!!!!!!!!!!!!!!help meeeeeeeeeeeeee pleaseeeeeeeeeee rn rnnnn!!!!!!!!!!!!!!!!!!!!!!!!!!!!!!!!!!!!!!!!!help meeeeeeeeeeeeee pleaseeeeeeeeeee rn rnnnn!!!!!!!!!!!!!!!!!!!!!!!!!!!!!!!!!!!!!!!!! WILL GIVE BRAINIEST!!!! Chris earns $298.65 each week. The federal government withholds 17% ofthat for federal income tax. How much is withheld from her pay annuallyfor federal income tax?a. $15,529.80b. $50.77c. $2,640.07d. $2,963.77